Sunteți pe pagina 1din 40

lSAT 101: CORE CONCEPTS

10 Order Games
Ouestions 1-7

4.

A closet contains exactly six hangers-I. 2, 3, 4, 5, and


6-hanging, in that order, from left to right. It also contains
exactly six dresses-one gauze, one linen, one polyester, one
rayon, one silk, and one wool-a different dre ss on each of
the hangers, in an order satisfying the following conditions:
The gauze dress is on a lower-numbered hanger than the
polyester dress .
The rayon dress is on hanger 1 or hanger 6.
Either the wool dress or the silk dress is on hanger 3.
The linen dress hangs immediately to the right of the silk
dress .
I.

(8)
(C)
(D)
(E)

2.

I: wool; 2: gauze ; 3: silk; 4: linen; 5: polyester;


6: rayon
I: rayon; 2: wool; 3: gauze; 4 : silk; 5: linen;
6: polyester
I: polyester; 2: gauze ; 3: wool; 4: silk; 5: linen;
6: rayon
1: linen; 2: silk; 3: wool ; 4: gauze; 5: polyester;
6: rayon
I: gauze; 2: rayon; 3: silk ; 4: linen ; 5: wool ;
6: polyester

If both the silk dress and the gauze dress are on odd

numbered hangers . then which one of the following

could be true?

(A)
(B)
(C)
(D)
(E)

3.

The
The
The
The
The

polyester dress is on hanger I.


wool dress is on hanger 2.
polyester dress is on hanger 4.
linen dress is on hanger 5.
wool dress is on hanger 6.

If the silk dress is on an even-numbered hanger,

which one of the following could be on the hanger

immediately to its left?

(A)
(8)
(C)
(D)
(E )

(A) The silk dress is on hanger I.


(8) The wool dress is on hanger 3.
(C) The linen dress is on hanger 4.
(D) The linen dress is on hanger 5.
(E) The rayon dress is on hanger 6.
5.

the
the
the
the
the

gauze dress
linen dress
polyester dress
rayon dress
wool dress

B The Princeton Review. Inc .

Which one of the following CANNOT be true ?


(A) The linen dress hangs immediately next to the
gauze dress.
(8) The polyester dress hangs immediately to the right
of the rayon dress.
(C) The rayon dress hangs immediately to the left of
the wool dress.
(D) The silk dress is on a lower-numbered hanger than
the gauze dress.
(E) The wool dress is on a higher-numbered hanger
than the rayon dress.

Which one of the foll.owing could be an accurate

matching of the hangers to the fabrics of the dresses

that hang on them?

(A)

If the polyester dress is on hanger 2, then which one of


the following must be true?

6.

Which one of the following CANNOT hang


immediately next to the rayon dress?
(A) the gauze dress
(8) the linen dress
(C) the polyester dress
(D) the silk dress
(E) the wool dress

7.

Assume that the original condition that the linen dress


hangs immediately to the right of the silk dress is
replaced by the condition that the wool dress hangs
immediately to the right of the silk dress. If all the other
initial conditions remain in effect, which one of the
following must be false?
(A)
(8)
(C)
(D)
(E)

The
The
The
The
The

linen dress is on hanger I.


gauze dress is on hanger 2.
wool dress is on hanger 4.
silk dress is on hanger 5.
polyester dress is on hanger 6.

LSAT 101; CORE CONCEPTS

10.
There are
six groups in this year's Civic Parade:
firefighters, gymnasts, jugglers, musicians, puppeteers, and
veterans. Each group marches as a unit; the groups are
ordered from first, at the front of the parade, to sixth, at the
back. The following conditions apply:
At least two groups march behind the puppeteers but
ahead of the musicians.
Exactly one group marches behind the firefighters but
ahead of the veterans.
The gymnasts are the first, third, or fifth group.
8.

9.

If the gymnasts march


ahead of the
veterans, then which one of the following could be the
fourth group?
(A) gymnasts
(B) jugglers

musicians

puppeteers

(E) veterans

12

(A) firefighters
(B) gymnasts

(C)
(D) musicians

veterans

11. . If the jugglers are the fifth group, then which one of the
following must be true?
(A) The puppeteers are the first group,
(B) The
are the first group.

Which one of the following could be an accurate list


of the groups in the Civic Parade in order from first to
last?
(A) firefighters, puppeteers, veterans, musicians,
gymnasts, jugglers
gymnasts, puppeteers, jugglers, musicians,
firefighters, veterans
veterans, puppeteers,
gymnasts,
jugglers, musicians
puppeteers, gymnasts, firefighters
musicians, veterans
(E) musicians, veterans, jugglers, firefighters,
gymnasts, puppeteers

I ~ The Prince.on Aeyiew,

lnc.

If the veterans march immediately behind the


puppeteers, then which one of the following could be
the second

(C) The veterans are the second group.


(D) The gymnasts are the third group.

(E) The musicians are the sixth group.


12,

Which one of the follOWing groups CANNOT march


immediately behind the gymnasts?
(A)
(B)
(C)
(D)

firefighters
jugglers
musicians
puppeteers

veterans

UNIT 1

10 Order Games
3.

Questions 1-5
Six racehorses-K, L, M, N, 0, and P-will be assigned
to six positions arranged in a straight line and numbered
consecutively I through 6. The horses are assigned to the
positions , Olle horse per position, accord ing to tile following
conditions:
K and L must be assigned to positions that are separated
from each other by exactly one position.
K and N cannot be assigned to positions that are next to
each other.
N must be assigned to a higher-numbered position
than M .
P must be assigned to posi ti on 3.
1.

Which one of the following lists an acceptable

assignment of horses to positions 1 though 6,

respectively ?

(A)
(B)
(C)
(D)
(E)

2.

K, L, P, M , N,
M. K, P, L, N,
M. N , K, P, L,
N , 0 , P, K, M,
0, M, P, L, N,

0
0
0
L
K

Which one of the following is a complete and accurate


list of the positions any one of which ca n be the
position to which K is ass igned ?
(A)
(B)
(C)
(D)

(E)

L 2
2,3

Which one of the following CANNOT be true?


(A)
(B)
(C)

(D)
(E)
4.

Which one of the following must be true?


(A)
(B)
CC)
(D)
(E)

5.

K is assigned to position 2.
L is ass igned to position 2.
M is assigned to position 1.
M is assigned to position 5.
0 is assigned to position 2.

Either
Either
Either
Either
Either

K or else L is assigned to position 2.


K or else L is assigned to position 4.
M or else N is assigned to position 2.
M or else N is assigned to position 5.
M or else 0 is assigned to position 6.

Which one of the following CANNOT be true?


(A) Land N are assigned to positions that are next to
each other.
(B) M and K are assigned to positions that are next to
each other.
(C) M and 0 are assigned to positions that are nex t to
each other.
CD) L and N are assigned to positions that are
se parated from each other by exactly one
posi tion .
(E) M and P are assigned to posi tion s that are
separated from each other by exactly one
posilion.

')
Ll
w, .

2,4,5
2, 4, 6

The Princeton Review, In c

I 25

lSAT 101, CORE CONCEPTS

8,

If car 4 is

which one of the following must be

true?
Six cars are to be
in a straight line, and will be
numbered 1 through 6, in order, from the front of the line to
the back of the line, Each car is exactly one color: two are
green, two are orange, and two are purple, The arrangement
of cars is restricted as follows:
No car can be the same color as any car next to it in line,

Either car 5 or car 6 must be purple.

Car I cannot be orange,

Car 4 cannot be green,

6.

The cars in which one of the


be the same color as each other?

(A) Car I is orange.


(B) Car 2 is green,

Car 3 is orange.

(D) Car 5 is green,


(E) Car 6 is purple.

9.

Car
Car
(C) Car
(D) Car
Car

CANNOT

(A) cars 1 and 4


(B) cars I and 5
(C) cars 3 and 5
(D) cars 3 and 6

cars 4 and 6

7,

If car 2 is the same color as car 4, then which one of


the followinl! statements must be true')
(A) Car 1

Car
(C) Car
(D) Car
(E) Car

26 I

2 is orange.

3 is green,
5 is
6 is green

The Pnnceton qevlew. Inc

Which one of the following statements must be false?

10,

2 is
4 is
5 is
6 is
6 is

green.

orange.

orange,

green.

If one of the two orange cars is


a third
green car, and if the arrangement of cars in line must
conform to the same restrictions as before, then which
one of the following is a complete and accurate list of
the cars each of which must be
car
car
(C) car
car
car

1, car 3

I, car 3, car 5

UNIT 1

14.
Seven consecutive time slots for a broadcast, numbered in
order 1
7, will be filled by six song
lpes~'-', H, L, 0, P, S-,-and
one news tape. Each
to a different time sIaL and no tape is
tape is to be
longer than any other tape. The broadcast 1S subiect to the
following restrictions:
L must be
before O.

The news tape must be


at some time after L.

There must be exactly two time slots between G and P,

of whether G comes before P or whether G


comes after P.

If P is played fifth, L must be played


(B)

(el

eE)
15,

first
second

third

fourth

sixth

What is the maximum number of tapes that can separate


S from the news')
CA) 1
(B) 2

3
(D) 4

second, which one of the following tapes


third'!

II. If G is

16.

(A) the news

(D) 0
(E) S
12.

~co~

17.

iliird

(C) fuurth
(D) fiM
(E) sixth

13.

the
the
(C) the
(D) the
(El the
(Al
(El

The news tape can be


in anyone of the
time slots EXCEPT the
(Al

Which one of the


which L can be played?

If Hand S are to be scheduled as far from each other


as
then the first, the second, and the third time
slots could be filled, "~0~d~';"~
(A) G, H, and
(B) S, G, and
(C) H, G, and
(D) H, L. and
(E) L, 0, and

is the latest time slot in

third

fourth

fifth
sixth

seventh

The time slot in which 0 must be


is
to which one of the
determined if G is
time slots'!
CA) the
the
the
(0) the
the

first

third

fourth

fifth

sixth

the news
L
0

The Princeton Review. Inc.

I 27

LSAT 101; CORE CONCEPTS

21.
A
instructor will schedule
one lesson for each of
six students-Grace,
Janet, Steve, Tom, and Una-one
lesson per
for six consecutive days. The schedule must
conform to the following conditions:
lesson is later in the schedule than Janet's lesson.
Una's lesson is later in the schedule than Steve's lesson.
Steve's lesson is
three
after Grace's lesson.
Janet's lesson is on the first day or else the third

(A) first day


(B) second day

third day

fourlh

fifth day

22.

IS.

If Janet's lesson is scheduled for the first


then the
lesson for which one of the following students must be
scheduled for the sixth day?
(A) Grace
Steve

Tom

(E) Una
For which one of the following students is there an
acceptable schedule in which the student's lesson is on
the third
and another
schedule in which
the student's lesson is on the fifth day?
(A) Grace

(8)
(e) Steve
(D) Tom

Una

20.

Which one of the


is a complete and accurate
list of the students anyone of whom could be the
student whose lesson is scheduled for the second day?
(A) Grace
(8) Tom
Grace, Tom

Tom

(E) Grace, Henry, Tom

28 I

Prir.ceton ReView, loe

If Janet's lesson
scheduled for the third
one of the following could be true?

which

(A) Grace's lesson is scheduled for a later


than
lesson.
Grace's lesson is scheduled for a later day than
Una's lesson.
Henry's Jesson is scheduled for a later day than
Una's lesson.
(D) Tom's lesson is scheduled for a later day than
lesson.
(E) Tom's lesson is scheduled for a later day than
Una's lesson.

(8)

19.

If
lesson is scheduled for a
either
immediately before or immediately after Tom's lesson,
then Grace's lesson must be scheduled for the

23.

Which one of the following is a complete and accurate


list of
anyone of which could be the day for
which Torn's lesson is scheduled?
first, second, third
second, third, fourth
second, fifth, sixth
fi rst, second, third, fourth
(E) second, third, fourth, sixth

UNIT 1

Questions 24-28

26.

Exactly seven products-P, Q, R , S, T, W, and X-are each to


be advertised exactly once ill a section of a catalog. The order
in \vhich they will be displayed is governed by the following
condition s :
Q must be di splayed in some position before W.
R must be displayed immediately before X.
T cannot be di splayed immediately before or immediately
after W.
S must be displayed either fir st or seventh.
Either Q or T must be displayed fourth.
24.

27.

(A) Q is displayed fifth.

(D) W is displayed third.


(E) X is displayed fifth.

If R is displayed sixth , then which one of the following


mu st be displayed

If X is displayed immediately before Q, then which one


of the following could be

Which one of the following could be true?


(B) Q is displayed seventh.
(C) R is displayed third.

28.

CD)

25

(A) R
(B) S
(C) T
CD) W
(E) X

Which one of the following CANNOT be the product


that is di splayed first ?
(A) P
(B) Q
(C) R
T
(E) X

If P is displayed second, then which one of the


following could be displayed third')

true~

fifth~

(A) P
(B) Q
(C) T
(D) W
(E) X

(A ) T is displayed first.
(B) R is di splayed fifth.
(C)
Q is displayed last

CD) Q is di splayed second.

(E) P is displayed second.

The Princeton Review, Inc

I 29

....

UNIT2

More 10 Order Games


4.

Q.yestions 1-7
During a period of six co nsecLl tive days-day 1 through da y
6-each of exactly six factories-F, G, H , J, Q, and R-will
be inspected . During this period , each of the factories wi ll be
inspected exactly once, o ne factory per day. The sched ule for
the inspections must co nform to the following conditions:
F is inspected on either day 1 or day 6.
J is inspected on an earlier day than Q is inspected.
Q is inspected on the day immediately before R is
inspected.
If G is inspected on day 3, Q is in spected on day 5.

(A) G and H
(B) G a nd R
(C) Hand G

(D) Rand J
(E) Rand H

5.

factories in the order of their sc heduled inspections ,

from day 1 through day 6?

(A) F, Q , R, H , J, G

2.

Q,
H,
R,
Q,

R, F
R, F
F, H
R, F

Which one of the following must be fal se?


(A)
(B)
(C)
(D)
(E)

3.

G, H , J,
G, J, Q.
G, J, Q,
J, H , G,

The
The
The
The
The

inspecti o n
inspection
inspection
in spection
in spection

of
of
of
of
of

6.

G is scheduled for day 4.


H is scheduled for day 6.
J is scheduled for day 4.
Q is scheduled for day 3.
R is scheduled for day 2.

of ei th er G or H is scheduled for
of either G or J is sc heduled for
of e ither G or J is sched uled for
of either H or J is scheduled for
of either H or J is scheduled for

If the inspection s of G and of H are scheduled, not


necessaril y i n that order, for days as far apart as
possible, wh ich one of the following is a complete and
accurate list of the factories anyone of which co uld be
sched uled for inspection for day I?
(A) F. J

(B)
(C)
(D)
(E)

The in spection of which one of the following CANNOT


be scheduled for day 5')
(A) G
(B) H
(C) J

If the inspection of R is sched uled for the day


immediately before the inspection of F, which one of
the following must be true about the schedule?
(A) The inspection
day 1.
(B ) The in spection
day J .
(C) The inspecti o n
day 2.
(D) The inspection
day 3.
(E) The inspection
day 4.

I . Which one of the fo llow ing could be a list of the

(B)
(C)
(D)
(E)

The inspections scheduled for day 3 and day 5,


respectively, could be those of

G, H
G, H, J
F, G, H
F, G, H, J

(D) Q

If the inspection of G is scheduled for the day


immediately before the inspecti on of Q, whi ch o ne of

(E) R

the following co uld be true?

7.

(A)
(B )
(C)
(D)
(E)

The
The
The
The
The

inspection
inspection
inspecti on
inspection
inspection

of
of
of
of
of

G is sched uled for day 5.


H is scheduled for day 6.
J is sched ul ed for day 2.
Q is schedu led for day 4.
R is sched uled for day 3.

The

Princeton Review, Inc . \

65

tSAT 101: CORE CONCEPTS

Questions 8-13

11.

During a period of seven consecutive days-from day 1


through day 7-seven investors-Fennelly, Gupta, Hall. Jones,
Knight, L6pez, and Moss-will each view a building site
exactly once. Each day exactly one investor will view the
site. The investors must view the site in accordance with the
following cond itions:
Fennelly views the site on day 3 or else day 5.

Lopez views the site on neither day 4 nor day 6.

If Jones views the site on day 1, Hall views the site on

day 2.
If Knight views the site on day 4, L6pez views the site on
day 5.
Gupta views the site on the day after the day on which
Hall vIews the site.
8.

(A)
(B)
(C)
(D)
(E)
12.

9.

If Jones views the site on day 1. which one of the


following investors must view the site on day 4?
(A)
(B)
(C)
(D)
(E)

10.

(D) 4
(E) 5

66 I

Fennelly
Gupta
Knight
L6pez
Moss

Jones
Jones
Jones
Moss
Moss

views
views
views
views
views

The Princewn Review, Inc .

13 .

If the day on which Gupta views the site and the day
on which L6pez views the site both come at some time
before the day on which Fennelly views the site, which
one of the following is an investor who could view the
site on day 3?
(A) Fennelly

(B)
(C)
(D)
(E)

If Knight views the site on day 4 and Moss views the


site on some day after the day on which Jones views the
site, which one of the following must be true?
(A)
(B)
(C)
(D)
(E)

If Hall views the site on the day after the day Knight
views the site and if Fennelly views the site on the day
after the day Lopez views the site. then Jones must view
the site on day
(B) 2
(C) 3

(A) Hall, Gupta, Fennelly, Moss, Knight, L6pez , Jones

Hall. Gupta, Lopez, Fennelly. Moss, Knight, Jones


L6pez. Gupta, Hall, Moss, Fennelly, Jones, Knight
L6pez. Jones , Fennelly, Knight, Hall Gupta, Moss
Lopez, Jones, Knight, Moss, Fennelly, Hall, Gupta

the
the
the
the
the

site
site
site
site
site

on
on
on
on
on

day
day
day
day
day

I.
2.
6.
2.
6.

Knight
Moss
Jones, Moss
Knight, Moss
Jones, Knight. Moss

(A) 1

Which one of the following co uld be the order in which


the investors view the site, from day 1 through day 7?
(B)
(C)
(D)
(E)

If Hall views the site on day 2, which one of the


following is a complete and accurate list of investors
anyone of whom could be [he investor who views the
site on day 4')

Gupta
Jones
Knight
Moss

UNIT 2

16.

Qyestions 14-18
During a single week , from Monday through Friday, tours will
be conducted of a company's three divisions-Operations,
Production , Sales. Exactly five tours will be conducted that
week, one each day. The schedule of tours for the week must
conform to the following restrictions:
Each division is toured at least once.
The Operations division is not toured on Monday.
The Production division is not toured on Wednesday.
The Sales division is toured on two consecutive days, and
on no other days.
If the Operations division is toured on Thursday, then the
Production division is toured on Friday.

14.

15 .

CA)
(B )
(C)
(D)
(E)
17.

Which one of the following CANNOT be true of the


week 's tour schedule?
(A) The division that is toured
toured on Tuesday.
(B) The division that is toured
toured on Friday.
(C) The division that is toured
toured on Thursday.
CD) The division that is toured
toured on Friday.
(E) The division that is toured
toured on Friday.

on Monday is also
on Tuesday is also
on Wednesday is also

If in addition to the Sales division one other division is


toured on two consecutive days , then it could be true of
the week 's tour schedule both that the
(A) Production division is toured on Monday and that
the Operations division is toured on Thursday
(B) Production division is toured on Tuesday and that
the Sales division is toured on Wednesday
(C) Operations division is toured on Tuesday and that
the Production division is toured on Friday
(D) Sales di vision is toured on Monday and that the
Operations division is toured on Friday
(E) Sales division is toured on Wednesday and that
the ProductlOn division is toured on Friday

Monday
Tuesday
Wednesday
Thursday
Friday

If in the week 's tour schedule the division that is toured


on Monday is not the division that is toured on Tuesday,
then which one of the following could be true of the
week 's schedule?
(A) A tour of the Sales division is scheduled for some
day earlier in the week than is any tour of the
Production division.
(B ) A tour of the Operations division is scheduled for
some day earlier in the week than is any tour of
the Production division.
(C) The Sales division is toured on Monday.

CD) The Production division is toured on Tuesday.

(E) The Operations division is toured on Wednesday.

on Monday is also

on Thursday is also

If in the week's tour schedule the division that is toured


on Tuesday is also loured on Friday, then for which
one of the following days must a tour of the Production
division be scheduled?

18 .

If in the week's tour schedule the division that is toured


on Tuesday is also toured on Wednesday, then which
one of the following must be true of the week's tour
schedule?
(A)
(B)
(C)
(D)
(E)

The
The
The
The
The

Production division is toured on Monday.


Operations division is toured on Tuesday.
Sales division is toured on Wednesday.
Sales division is toured on Thursday.
Production division is toured on Friday.

The Prmceton Review, Inc

I 67

lSAT 101: CORE CONCEPTS

21
Each of seven television programs-H, J, L, P, Q, S, V-is
a different rank: from first
seventh (from
most popular to least popular). The ranking is consistent with
the
conditions:
J and L are each less popular than H.
] is more popular than Q,
S and V are each less popular than L
P and S are each less popular than Q,
S is not seventh,

19,

Which one of the

(A)],
(B) H,
(Cl H,
(D) H.
(E) H,

20,

L, Q, V,
L, Q, J, S,
J, Q, L, S,
1, V, L, Q,
L, V, 1,

If J is more popular than L, and S is more popular than


P, then which one of the following must be true of the
(A) J
second.
(B) 1 is third.
L is third.
Q is third,
P is seventh.

68 \

The Princeton Review, Inc.

(Al L
(B) J
(C) Q

22.

could be the order of the


to least popular?

S, P
P. V
V, P
S, P
P, S

Which one, of the following programs CANNOT be


ranked third?

than Q and J is less


than
L, then which one of the following could be true of the

If V is more

(A) P is more popular than S.

S is more popular than V,

(C) P is more popular than L

] is more popular than V,

(E) Q more popular than V

23.

If Q is more
than L, then each of the
must be true of the ranking EXCEPT:
(A) H
first.
(B) L is fOUlth.

V is not fourth,

(D) ] is not third,

Q is third.

UNIT2

26.

Ql!estions 24-28
Five people-Harry, Iri s, Kate, Nancy, a nd Victor-are to be
scheduled as contestants on a television show, one contestant
per day, for five consecutive days from Mond ay through
Friday. The foll owing restrictions governing the scheduling of
contestants must be observed:
Nancy is not scheduled for Monday.
If Harry is scheduled for Monday, Nanc y is scheduled for
Friday.
If Nancy is scheduled for Tuesday, Iris is scheduled for
Mond ay.
. Kate is scheduled fo r the nex t day after the day for which
Victor is scheduled.

24.

25.

(A) Iri s is scheduled for Friday.


(B) Nancy is scheduled for Tuesday.
(C) Nancy is scheduled for an earlier day than the day
for which Harry is scheduled .
(D) Nancy is scheduled for an earlier day than the day
for which Iris is sc heduled.
(E) Nancy is scheduled for an ea rlier day than the day
for which Kate is scheduled.
27.

Monday
Tuesday
Wednesday
Thursda y
Frid ay

If Iris is scheduled for the next day after Harry, which


one of the following lists all [hose day s anyone of
which could be the day for whi ch Harry is scheduled?
(A)
(B)
(C )
(D)
(E)

Monday,
Monday,
Monday,
Monda y,
M onday,

Tuesday
Wednesd ay
Thursday
Tuesday, Wednesday
Wednesday, Thursday

If Kate is scheduled for Friday, whi ch one of the


fo llowing mu st be true ?
(A)
(B)
(C)
(D)
(E )

Victor can be scheduled for any day EXCEPT


(A)
(B)
(C)
(D)
(E)

If Kate IS sc heduled for Wednesday, which one of the


following could be true?

28.

Harry is scheduled for Tuesday.


Harry IS scheduled for Wednesda y.
Iri s is scheduled for Monday.
Iris is scheduled for Wednesday.
Nancy is scheduled for Wednesday.

If Iris is scheduled for Wed nesd ay, whi ch one o f the


following must be true ?
(A) H arry is scheduled for a n earlier da y than the day
for which Nancy is schedul ed.
(B ) Harry is scheduled for an earlier day than the da y
for which Kate is scheduled .
(C) Kate is scheduled for an earlier day than the day
for which Harry is sc hedul ed.
CD) Nancy is scheduled for an earlier da y than the day
for whi ch Kate is scheduled .
(E) Nancy is scheduled for an earli er day than the day
for which Iris is schedul ed .

The Prin ce ton Re VIew. In c

I 69

l SAT 101; CORE CONCEPTS

20 Games: Spatial Arrangements


Questions 1-5
There are exactly ten stores and no other buildings on Oak
Street. On the north side of the street, from west to east, are
stores 1, 3, 5, 7, and 9; on the south side of the street, also
from west to east, are stores 2, 4, 6, 8, and 10, The stores
on the north side are located directly across the street from
those on the south side, facing each other in pairs, as follows:
1 and 2; 3 and 4; 5 and 6; 7 and 8; 9 and 10. Each store is
decorated with lights in exactly one of the following colors:
green, red, and yellow. The stores have been decorated with
lights according to the following conditions:
No store is decorated with lights of the same color as
those of any store adjacent to it.
No store is decorated with lights of the same color as
those of the store directly across the street from it.
Yellow lights decorate exactly one store on each side of
the street.

Red lights decorate store 4.

Yellow lights decorate store 5.

1.

Which one of the following could be an accurate list of


the colors of the lights that decorate stores 2, 4, 6, 8,
and 10, respectively?
(A)
(B)
(C)
(D)
(E)

2.

If green lights decorate store 7, then each of the

following statements could be false EXCEPT:

(A)
(8)
(C)
(D)
(E)

84

green , red, green, red, green


green, red, green, yellow, red
green, red, yellow, red, green
yellow, green, red , green, red
yellow, red, green , red, yellow

Green lights decorate store 2.


Green lights decorate store 10.
Red lights decorate store 8.
Red lights decorate store 9.
Yellow lights decorate store 2.

I The Princeton Review, Jnc .

3.

Which one of the following statements must be true?


(A)
(B)
(C)
(D)
(E)

4.

Green lights decorate store 10.


Red lights decorate store I.
Red lights decorate store 8.
Yellow lights decorate store 8.
Yellow lights decorate store 10.

If green lights decorate five stores on the street, then


which one of the following statements must be true')
(A) Green lights decorate store 9.
(8) Red lights decorate store 2.

(C) Red lights decorate store 7.


(D) Red lights decorate store lO.
(E) Yellow lights decorate store 8.
5.

Suppose that yeJl.ow lights decorate exactly two stores,


not just one, on the south side of the street and decorate
exactly one store on the north side. If all of the other
conditions remain the same, then which one of the
following statements must be true')
(A) Green lights decorate store l.
(8) Red lights decorate store 7.

(C) Red lights decorate store 10.


(D) Yellow lights decorate store 2.
(E) Yellow lights decorate store 8.

lSAT 101: CORE CONCEPTS

Ouestions 6-10
A school has exactly four dormitories that are to be fully
occupied-Richards, Tuscarora, Veblen, and Wisteria-each
consisting entirely of a North wing and a South wing. The
following rules govern assignment of students to dormitory
wings:
Each wing is assigned only male students or only female
students.
Exactly three wings have males assigned to them.
Richards North and Tuscarora Nonh are assigned
females.
If a dormitory has males assigned to one of its wings,
then its other wing is assigned females.
If males are assigned to Veblen South, then Wisteria
North is assigned males.
6.

8.

(A) Richards South


(8) Wisteria South

(C) Tuscarora South


(D) Veblen South
(E) Veblen North
9.

(A) Richards North, Tuscarora North


(8) Richards North, Tuscarora North, Veblen North

(C) Richards North, Tuscarora North, Wisteria South


(D) Richards North, Tuscarora North, Veblen North,
Wisteria South
(E) Richards North, Richards South, Tuscarora North,
Veblen North, Wisteria South

North, then which one of the following could be two


other wings that are also assigned females?
(A) Richards North and Tuscarora South
(C) Richards South and Tuscarora North
(D) Tuscarora North and Wisteria South
(E) Tuscarora South and Wisteria South
7.

It CANNOT be true that females are assigned to both


(A) Richards South and Wisteria South
(8) Richards South and Tuscarora South

(C) Richards South and Veblen North


(D) Tuscarora South and Wisteria South
(E) Veblen North and Wisteria South

86

I The Princeton Review, Inc

If males are assigned to Veblen South, whi.ch one of the


following is a complete and accurate list of the wings
that CANNOT be assigned males?

If females are assigned to Veblen South and Veblen

(8) Richards South and Wisteria South

If Wisteria North is assigned females, then females must


also be assigned to which one of the following?

10.

If Tuscarora South is assigned females, then it could be


true that females are assigned to both
(A) Richards South and Wisteria North
(8) Richards South and Wisteria South

(C) Veblen North and Wisteria North


(D) Veblen South and Wisteria South
(E) Veblen South and Veblen North

lSAT 101 : CORE CONCEPTS

2D Games: Spatial Arrangements


2.

Questions 1-6

(A )
(B)
(C)
(D )
(E)

Six c ities are located within the numbered areas as follows:

City

City
l

City

3.

City
4

4.
City

City

Within the six-city area there are exactly four hos pitals. two
jails. and two universities. These eight institutions are located
as follows:
No institution is in more than one of the cities.
None of the cities contains more than one jail , and none
contains more than one university.
None of the cities contains both a jail and a uni versity.
Eac h jail is located in a city that contains at least one
hospital.
The universities are located in two cities that do not share
a common boundary.
City 3 contains a university, and city 6 contains a jail.

1.

Which one of the following could be tru e?


(A )
(B)
(C)
(D)
(E)

100 I

City
City
City
City
City

S contains
6 contains
2 co ntains
3 contains
3 contains

The Prince ton Revi ew, Inc

a
a
a
a
a

univ ersity.
uni versity.
jail.
Jail.
hospital.

Which one of the following could be true?

(A)

1, 4

(B)
(C)
(D )
(E)

2, 4
4, S

exactly
exactly
exactly
exac tly
exactly

one
one
one
one
one

hospital.
university.
jail.
universit y.
university.

1, 4, S
I , 2, 4, S

If each of the six cities contains at leas t one of the eight


institutions, then which one of the following mu st be
true ?
There
There
There
There
There

is
is
is
is
is

a
a
a
a
a

jail in cit y
hospital in
hospital in
hospital in
jail in city

l.
city 2 .
city 3.
city 4.
4.

In which one of the following cities must there be fewer


than three hospital s?
(A )
(B)
(C)
(D)
(E )

6.

l co ntains
I contains
2 contains
S contains
6 contains

Which one of the following is a complete and accurate


list o f the cities anyone of which could contain th e jail
that is not in city 6 ?

( A)
(B)
(C)
(D)
(E)

S.

City
City
City
City
City

1
2
4

S
6

If one of the cities contains exac tl y two hospitals and


exactly one university, then which one of the following
lists three cities that might, among them , contain no
hospital ?
(A) I ,3, S
(B) I, 4, S
(C)2, 3, S
(D) 2, 4,6
(E) 4, S, 6

"

UNIT3

9.

Qy.estioJls 7- 12
On an und eveloped stree t, a developer will simulta neou sly
build four houses on one side, numbered consecutively I,
3, 5, and 7, and four houses on the opposite side, numbered
consecutive ly 2, 4, 6, and 8. Houses 2 , 4, 6, and 8 will face
houses 1, 3, 5, a nd 7, respectively. Each house will be exaclly
one of three styles-ranch , split-leveL or Tudo r-according to
the following conditions:
Adjacent houses are of different styles.

No split-level house faces another split level hou se.

Every ra nch ho use has at leas t o ne Tudor house adjacent

to it.

House 3 is a ra nch house.

House 6 is a split-level house.

7.

Any of the following could be a TUdor house EXCEPT


house

(A)
(B)
(C)
(D)
(E )
10.

11.

(C) 4
(D) 7
(E) 8

If th ere is one ranch house di rectly opposite another


ranch house, which one of the following could be true?
(A)
(B)
(C)
(D)
(E)

House
House
House
House
House

8
7
4
2
1

is
is
is
is
is

a
a
a
a
a

ranch ho use.
split-level house.
Tudor house.
split-level house.
ranch hou se.

12.

is
is
is
is
is

a
a
a
a
a

Tudor ho use
Tudor ho use
ranch house
Tudor house
ran ch house

one ranch house


o ne Tudor house
two Tudor houses
four ranch houses
fi ve ran c h houses

If no hou se faces a house of the same style. then it mu st


be true that hou se
(A)
(B)
(C)
(D)
(E )

(B) 2

1
2
5
7
8

On the street, there could be exactly


(A)
(B)
(C)
(D)
( E)

(A) 1

8.

If house 4 is a Tudor house, thell it could be tru e that

house

1 is
1 is
2 is
2 is
4 is

a
a
a
a
a

split-level house
Tudo r house
ranch house
split-level house
Tudor house

If the condition requiring house 6 to be a split level


house is suspended but all other original conditions
remain th e same, th en any of the following could be
an accurate li st of the styles of hou ses 2, 4, 6, and 8,
respectively, EXCEPT:
(A)
(B )
(C)
(D)
(E)

ranch , split-level, ran c h. Tudor


split-level, ranch , Tudor, split-level
split-level, Tudor, ranch, split-level
Tudor, ran ch, Tudor. split-level
Tudor, split-level, ranch, Tud or

The Prin ce ton Review, Inc.

\101

LSAT 101: CORE CONCEPTS

IS,
J-and two
and M-will be
six stalls, one animal per stall. The stalls
as follows:
First Row:
1 2 3
Second Row:
4 5 6
The
stalls that face each other are stalls 1 and 4, stalls 2
and 5, and stalls 3 and 6. The following conditions
The tigers' stalls cannot face each other.

A lion must be
to stall J.

H must be assigned to stall 6.

J must be assigned to a stall numbered one higher than

K's stall.
. K cannot be assigned to the stall that faces H's stall.

If K's stall is in the same row as H's stall, which one of


the following must be true?
(A) F's stall is in the same row as 1's stalL
(B) F is
10 a lower-numbered stall than G.
G is
to a lower-numbered stall than M .
G's stall faces H's stall.
(E) M's stall is in the same row as G's stall.

17.

Th. Prine.wn Review. Inc,

If J is
to stall 3, which one of the following
could be true')
to stall 2.
F is
(B) F is assigned to stall 4.

G is assIgned to stall I.

G is
to stall 4.

(E) M
ass;,med to stall 5.

Which one of the following could be true?


than 1's stall.

F's stall is numbered one


H's stall faces M's stall.

J is assigned to stall 4.

(D) K's stall faces 1'5 stall.


(E) K's stall is in a different row than 1's stall.

102

16.

Which one of the following must be true?


(A) F is
to an even-numbered stalL
F is assigned to stall 1.
J is assigned to stall 2 or else stall
CD) J is
to stall 3 or else stall 4.
(El K is assigned to stall 2 or else stall 4.

14.

must be true?

10 stall 2,
A tiger is
A
is
to stall 5.
(C) K's stall is in a different row from M's stall.
(D) Each
is assigned to an even-numbered stall.
(E) Each lion is assigned to a stall that faces a
stalL

Four lions-F, G,

13.

Which one of the

IS.

vVhich one of the following must be true?


A
(B) A
A
(D) A
(E) A

to stall 2.
tiger is
tiger is
to stall 4.

is assigned to stall 5.

lion is assigned to stall 3.


lion is assigned to stall 4.

.............-

UNlT 4

2D Games: Combining Order and Groups


4,

If Thea was born after Will was born, then how many
of the children, from firstborn to

Of the five Pohl children-Sara, Thea, Uma, Will, and


are left-handed and two
right-handed. Each of
the five children was born in a different one of seven calendar
1990
1996. The following conditions
No two left-handed children were born in
consecuti ve years.
No two
children were born in
consecutive years,

Sara, who is left-handed, was born before Uma,

Zoe was born before both Thea and Wi! L

A left-handed child was born in 1991.

Uma, who is right-handed, was born in 1991

1.

If Sara was born before Zoe was born, then which one

of the following statements CANNOT be true?

(A) Will is left-handed.


(B) Zoe is left-handed.
(e) Thea was born after Will was born.
(D) Uma was born after Zoe was born.

No child was born in 1990.

3.

(e) three
CD) four

five

5.

If none of the children was born in 1995. then which


one of the following statements must be true?
(A) Thea was born in 1994.

Will was born in 1994.

(e) Will was born in 1996.


CD) Zoe was born in 1990.

Zoe was bom in 1994,

Which one of the following could be an accurate

matching of each Pohl child with the year in which that

child was born?

(A) Sara: 1990; Zoe 1992; Uma: 1993; Will: 1994;


and Thea: 1995
(B) Sara: 1991; Uma: 1993; Thea: 1994; Zoe: 1995;
and Will: 1996
Zoe:
Sara: 1991; Uma: 1992; Thea: 1994;
and Will: 1995
(D) Zoe: 1990; Sara: 199]: Uma: 1993: Theo: 1994;
and Will: 1995
(E) Zoe: 1990; Sara: 1991; Uma: 1993; Thea: 1994;
and Will: 1996

2.

CA) one
(B) two

6.

If Theo is right-handed, then each of the following


statements must be false EXCEPT:
(A) Thea was born in 1996.

Will was born in 1995.

(C) Uma was born


three years before Thea
was born.
(D) Zoe was born exactly one year before Thea was
born.

Will is

7.

If Zoe was born before Uma was born, then which one
of the following statements must be false?
(A) No child was born in 1992.

No child was born in 1995.

eel Theo is left-handed.

Zoe is left-handed.

Will is left-handed.

Which one of the following must be false?


(A) None of the children was born in 1990, nor was a
child born in 1992.
None of the children was born in 1992, nor was a
child born in 1995.
None of the children was born in 1994, nor was a
child born in 1996.
One of the children was born in 1990, and another
in 1993.
(E) One of the children was born in 1993, and another
in 1995.

The Princetorl Review,

121

...--
UNIT 4

Questions 8-12
Maggie's Deli is open exactly five days every week: Monday
through Friday, Its staff, each of whom works on at least one
day each week, consists of exactly six people-Janice, Kevin,
Nan, Ophelia, Paul, and Seymour. Exactly three of them
Janice, Nan, and Paul-are supervisors, The deli's staffing is
consistent with the following:
Each day 's staff consists of exactly two people, at least
one of whom is a supervisor.
Tuesday's and Wednesday's staffs both include Ophelia,
Of the days Nan works each week, at least two are
consecutive,
Seymour does not work on any day before the first day
Paul works that week,
Any day on which Kevin works is the first day during the
week that some other staff member works,
8,

9,

(A)
(B)

(C)
(D)
(E)

to,

It.

Janice works Monday and Tuesday,


Kevin and Paul work Friday,
Seymour works Monday and Friday,
Janice and Kevin work Thursday,
Paul works Monday and Friday,

Which one of the following CANNOT be the pair of


staff that works Monday?
(A)
(B)
(C)
(D)
(E)

12,

Janice
Kevin
Nan
Paul
Seymour

Each of the following could be true EXCEPT:


(A)
(B)
(C)
(D)
(E)

Which one of the following could be an accurate

staffing schedule?

(A) Monday: Janice, Kevin

Tuesday: Nan, Ophelia

Wednesday: Nan, Paul

Thursday: Kevin , Paul

Friday: Janice, Seymour

(B) Monday: Paul, Seymour

Tuesday: Ophelia, Paul

Wednesday : Nan, Ophelia

Thursday : Kevin , Nan

Friday: Janice, Seymour

(C) Monday: Janice, Kevin

Tuesday: Nan, Ophelia

Wednesday : Nan, Ophelia

Thursday : Kevin, Paul

Friday : Paul , Seymour

(D) Monday: Janice, Kevin

Tuesday: Janice, Ophelia

Wednesday: Nan, Ophelia

Thursday: Nan, Seymour

Friday: Kevin, Paul

(E) Monday : Paul, Seymour

Tuesday: Ophelia, Paul

Wednesday: Nan, Ophelia

Thursday: Janice, Kevin

Friday: Nan, Paul

If Kevin and Paul work Thursday, who must work


Friday?

Janice and Seymour


Kevin and Paul
Paul and Seymour
Nan and Ophelia
Janice and Nan

Which one of the following could be true')


(A)
(B)
(C)
(D)
(E)

Nan works Wednesday and Friday only,


Seymour works Monday and Paul works Tuesday,
Kevin works Monday, Wednesday, and Friday,
Nan works Wednesday with Ophelia and Thursday
with Kevin,
Ophelia and Kevin work Tuesday,

The Princeton Re . . lew, Inc .

I 123

LSAT 101. COR ECO NCEPTS

2D Games: Combining Order and Groups


3.

Ouestions 1-6
Four boys-Fred, Juan , Marc, and Paul-and three girls
Nita, Rachel , and Trisha-will be assigned to a row of
five adjacent lockers, numbered consecutively 1 through 5,
arranged along a straight wall. The fol/owing conditions
govern the assignment of lockers to the seven children :
Each locker must be assigned to either one or two
children , and each child must be ass igned to exactly one
locker.
Each shared locker mus t be assigned to one girl and one
boy.
Juan must share a locker, but Rachel cannot share a
locker.

Nita's locker cannot be adjacent to Trisha's locker.

Fred must be assigned to locker 3.

l.

2.

If Trisha is assigned to locker 3 and Marc alone is

assigned to locker 1, then which one of th e following

must be true?

(A)
(B)
(C)
(D)
(E)

132

locker
locker
locker
locker
(E) locker
(A)
(B)
(C)
( D)

4.

Juan is
Juan is
Paul is
Rachel
Rac hel

assigned to locker 4.
assigned to locker 5.
assigned to locker 2.
is assigned to locker 2.
is assigned to locker 5.

I The Princeton Re vlew, lnc .

5.

one
two
three
four
five

If the first three lockers are assigned to girls, which one


of the following must be tru e?
(A)
(B)
(C)
(D)
(E)

6.

2
4
1, locker 2
1, locker 4
2, locker 4

Once Rachel has been assigned [0 a locker, what is the


maximum number of different lockers each of which
could be the locker to which Juan is ass igned?
(A)
(B)
(C)
(D)
(E)

Which one of the following is a complete and accurate

list of the children who must be among those assigned

to shared lockers?

(A) Fred, Juan


(B) Juan, Paul
(C) Juan, Marc, Paul
(D) Juan, Marc, Trisha
(E) Juan, Nita , Trisha

If the four boys are assigned to consecutively numbered


lockers and Juan is assigned to locker 5. then which
one of the following is a complete and accurate list of
lockers each of which CANNOT be a shared locker?

Juan is assigned to locke r l.


Nita is assigned to locker 3.
Trisha is assigned to locker 1.
Juan is assigned to the same locker as Trisha.
Paul is assigned to the same locker as Trisha.

If lockers 1 and 2 are each assigned to one boy and are


not shared lockers, then locker 4 must be assigned to

(A)
(B)
(C)
(D)
(E)

Juan
Paul
Rachel
Juan and Nita
Marc and Trisha

UNIT 11

10.

Juarez must evaluate X

(e) Juarez evaluates an advanced textbook during

week 2.
(D) Juarez evaluates Y before

G.
Juarez does no! evaluate any two introductory
textbooks

J1.

evaluated
Juarez,
(E) Y is the first of the advanced textbooks to be
evaluated
Juarez.
(e) Juarez does not evaluate any two introductory
textbooks in a row.
(D) Juarez evaluates G during week 5.
(E) Juarez evaluates Z during week 6.

Which one of the


is an
evaluation
schedule, with the textbooks listed in order of evaluation
from week 1 through week 6 ?

12,

Y, F, X, G, Z, H
(e) Juarez: G, H, F, X, Y, Z

If Juarez evaluates H
evaluates G

must be true?

week 3 and

(Al Juarez evaluates F


(B) Juarez evaluates G
(e) Juarez evaluates Z
week 1
(D)
evaluates X
(E) Rosenberg evaluates Y during week 5.
9.

Which one of the


is a
and accurate
list of those weeks
which Juarez must evaluate
an introductory textbook?
(A) week 1

X, G, R Y, Z, F

(D) Juarez: G, Z, F, X, H, Y
Z, F, X, G, Y, H
Juarez: H, Y, F, X, G, Z
Rosenberg: X, R Z, F, Y, G
8.

If
evaluates X
week 1 and F
week 2, which one of the following could be true?
(A) X is the third of the advanced textbooks to be

week 4.

(A) Juarez F, G, X, Z, H, Y

X, F, Z,
Y, H

Juarez: F, Y,
X, H, Z

must be true?

evaluates H
week 6.
evaluates an advanced textbook during

(A)

At a small press, six textbooks, three


G, and
three advanced-X, Y, and Z-will each be evaluated
once by the editor, Juarel, and once
the
six consecutive weeks-week 1
week 6, Each evaluator evaluates
one textbook per
week, No textbook will be evaluated
Juarez and
the same week, The following additional constraints
Rosenberg cannot evaluate any
textbook until
Juarez has evaluated that textbook.
Juarez eannot evaluate any advanced textbook until
has evaluated that textbook.
Rosenberg cannot evaluate any two introductory textbooks

Which one of the

(E) week
week
(D) week
(E) week
13.

1. week 5

L week
week 3
1, week 3, week 5

Whieh one of the following could be true?


(A) Juarez evaluates F
(B) Juarez evaluates Z
evaluates F
(e)

(D)

week 3.
evaluates H during week 2,
evaluates X durin week S.

If Juarez evaluates Z

week 2, then

must evaluate which one of the followil1i1 textbooks

during week 5 ?

(A) F
(B) H

(D) Y

co The Princeton Review, lJ)c 1133

tSAT 101: CORE CONCEPTS

16.
An art teacher will schedule exactly six of eight lectureslithography, naturalism, oils,
sculpture,
and watercolors-for three
2, and 3. There will
be
two lectures each day-morning and afternoon.
Scheduling is
by the following conditions:
2 is the only day for which oils can be scheduled.
Neither sculpture nor watercolors can be scheduled for the
afternoon.
can be scheduled for the same
Neither oils nor
as
If pastels is scheduled for
I or day 2, then the
lectures scheduled for the
immediately following
must be fresco and history, not necessarilv in
that order.

14.

15.

(A) fresco

naturalism

oils

(D)

(E)

17.

If
and fresco are scheduled for the
afternoons of day 2 and day 3,
which one
of the following is a lecture that could be scheduled for
the afternoon of day 1')

If oils and lithography are scheduled for the


of day 2 and
3,
which one of the
following CANNOT be scheduled for any day?
(A) fresco
(B) history
(C) naturalism
(D) pastels

Which one of the


is an acceptable schedule of
lectures for days I, 2, and 3, respectively?
(A) Morning: lithography,
sculpture
Afternoon: pastels, fresco, naturalism
(B) Morning: naturalism, oils, fresco
Afternoon: lithography, pastels, history
(C) Morning: oils, history, naturalism
Afternoon:
fresco, lithography
(D)
sculpture, lithography, naturalism
Afternoon: watercolors, fresco,
Morning:
fresco
Afternoon: lithography, his!Orv. naturalism

If lithography and history are scheduled for the


of day 2 and day 3,
which one
of the following lectures could be scheduled for the
morning of day I?

sculpture

18.

If neither fresco nor naturalism is scheduled for any day,


which one of the following must be scheduled for day I?
(B) lithography

oils

(D) pastels
(E) sculpture

19.

If the lectures scheduled for the


are fresco,
history, and
not
in that order,
which one of the following could be true')
(A)

(A) history
(B) oils
(C)
(D)

(E) watercolors

134

The

Review, Inc.

is scheduled for day 3.

(B) Naturalism is scheduled for day 2.


(C) Fresco is scheduled for the same day as

(D)

naturalism.
is scheduled for the same day as
naturalism.
History is scheduled for the same day as oils.

UNIT 4

Qilestions 20-25
During each of the fall , winter, spring, and summer seasons
of one year, Nikki and Otto each participate in exactly one of
the following five sports: hockey, kayaking, mountaineering,
running , and volleyball.
Each child participates in exactly four different sports
during the year.
In the fall, each child participates in mountaineering,
running, or volleyball.
In the winter, each child participates in hockey or
volleyball.
In the spring, each child participates in kayaking,
mountaineering, running, or volleyball.
In the summer, each child participates in kayaking,
mountaineering, or volleyball.
Nikki and Otto do not participate in the same sport
during the same season .
Otto's summer sport is volleyball.
20.

It CANNOT be true that both Nikki and Otto participate


during the year in which one of the following sports')
(A)
(B)
(C)
(D)
(E)

22.

Nikki ' s fall sport is running.


Nikki's winter sport is volleyball.
Nikki's spring sport is mountaineering.
Otto's fall sport is mountaineering.
Otto's spring sport is kayaking .

hockey
kayaking
mountaineering
running
volleyball

Which one of the following statements could be true ?


(A)
(B)
(C)
(D)
(E)

24.

25.

Nikki's fall sport is neither mountaineering nor


running.

Nikki's spring sport is neither mountaineering nor

running.
Nikki's summer sport is neither kayaking nor
mountaineering.
Otto's fall sport is neither mountaineering nor
running.
Otto's spring sport is neither kayaking, nor
mountaineering, nor running.

If Otto does not run during the year, then which one of
the following statements must be false ?
(A)
(B)
(C)
(D)
(E)

Which one of the following statements must be true ?


(A)
(B)
(C)
(D )
(E)

21 .

23.

Nikki' s fall sport is running.


Nikki's spring sport is running .
Nikki's summer sport is kayaking.
Otto's fall sport is mountaineering.
Otto's spring sport is kayaking.

Which one of the following statements could be true ?


(A) Nikki' s fall sport is mountaineering and Otto's
spring sport is running.
(B) Nikki's spring sport is running and her summer
sport is mountaineering.
(C) Nikki's spring sport is mountaineering and Otto's
fall sport is mountaineering .
(D) Nikki's spring sport is running and Otto's fall
sport is mountaineering .
(E) Nikki's summer sport is mountaineering and
Otto's spring sport is mountaineering.

If Nikki 's fall sport is running, then which one of the


following statements must be true')
(A)
(B)
(C)
(D)
(E)

Nikki ' s spring sport is kayaking .


Nikki's summer sport is mountaineering.
Otto's fall sport is mountaineering.
Otto's spring sport is kayaking.
Otto's spring sport is running.

The Princeton RevieW, lnc. l ll5

LSAT 101: CORE CONCEPTS

More Games: Mixed Bag


3
An official is
five
Ned, Olivia,
Patricia, and
parallel lanes numbered
I through 5. The official will also
each runner
to represent a different charity-F, G, H, J, and K-not
in order of the runner's names as given, The
restrictions apply:
K is
to lane 4,
to the only lane between the lanes of
F and G,
two lanes between Olivia's lane and the
lane of the runner
G.
is assigned to a higher numbered lane than the lane
to which Ned is
1.

Which one of the following is a


runners to lanes
the charity they

2.

;2
H

(8) G

(D)
(E)

2,

5.
J

K
K

K
K

140

I The Princeton Review, Inc

(E)
4,

Which one of the


is a
and accurate
list of runners each of whom could be the runner
F?
(A) Larry, Ned
(B) Patricia, Sonja

Ned, Olivia

(D)
Ned, Sonja

Ned, Patricia,

5,

J, then it must be true

If Ned is the runner


that

to lane 1
the runner
to lane 2
the runner
is the runner representing K
F

(D) Olivia is the nmner


H

Patricia is the runner


lA)

G
must be a lane

next to the lane to which Larry is


(B) next to the lane to which Ned is
separated by exactly one lane from the lane to
which Ned is
(D)
by

which Olivia is

(E) separated by

which

Lane
I
5

H
H

H
J

The lane to which Patricia is


that is

(A)
(B)

assif1.nment of

to lane 2, which one of the


must be made')

If Olivia is
following

6,

If Larrv represents J, which one of the following could


of runners to lanes?

1
(A) Larry
(B) Larry
(e)
(D) Ned
(E) Ned

;2

Olivia
Ned

Ned
Olivia
Patricia

Patricia

Olivia
Olivia

Ned
Patricia
Patricia

5.
Sonja
Patricia
Olivia
Sonja
Larry

UNIT 4

Ques tions 7- 12
A professor wi ll listen to exactly one speec h fro m each of
six stu deJ1ls -H. J. K. R, S , and T. T.he six speeches wi ll
be deli vered o ne at a time, consec utively, accordi ng to th e
fo llow ing conditions:
T he speeches deli vered by H , J, an d K, no m atter what
their order relati ve to eac h other, can not form a seq uence
of three consec uti ve speec hes.
T he speeches deli vered by R, S , and T, no matter what
their ord er relati ve to each o the r, can not fo rm a seq uence
of three consecuti ve speeches.
H 's speech must be ea rl ier th an S 's speec h.

1's speech can be ne ither fi rst nor sixth

Ts speech ca n be neither immed iately before nor

immediately after 1's speech.


7.

Whi c h one of the following could be th e order, from


first to last, in whic h the students deli ver their speec hes?
(A)
(B )
(C)
(D)
(E)

8.

If T delivers th e third speech, w hic h o ne o f the

fo llow in g must be true?

(A)
(B)
(C)
(D)
(E)

9.

H , J, R, S, T, K
H , R, T, K , S, J
K , J, T, H , S, R
R , J, K. T, H , 5
T, R, J, 5 , K , H

10.

If K delive rs the fi rst speec h and H deli vers the fifth


speec h, whi c h one of the follow in g must be true?
(A)
(B)
(C)
(D)
(E )

11 .

If R 's speech is immedi ate ly after 5' s speech and


immedi a tely before K's speec h, the n whi c h one of the
fo llowing could be true?
(A)
(B)
(C)
(D)
(E )

12.

R delivers the third speech.


T deli vers the fo urth speech .
1's speech is im med iately be fore H 's speec h.
K ' s speech is immediately before Ts speech.
R 's speech is immed iately before 1's speec h.

H 's
H 's
K 's
K 's
Ts

speech is
speec h is
speech is
speech is
speech is

immedi ately
immediately
immedi ately
immedi ate ly
immedi ately

before
before
be fore
before
before

5 's speec h.
T s speec h.
1's speech.
Ts speech.
S's speec h.

If K deli vers the tbird speech, any of the followi ng


could be the stud e nt who makes the fo urth speec h
EXCEPT

(A) H
(B ) J

(C) R
(D) 5
(E) T

H delivers the firs t speech .

J deli vers the fifth speech.


K delivers the fo urth speech .
R delivers the sixth speech.
S delivers the fo urth speech.

If S de li vers the thi rd speech and T deli vers the fo urth


speech, then whic h one of the fo llow ing must be true?
(A)
(B)
(C)
(D)
(E)

H delivers th e second speech.

J deli vers the fift h speech.


K de li vers the fi ft h speech.
K delivers the fi rs t speech.
R de livers the fi rst speech.

The Prince ton Review. Inc. 1

141

LSAT 101: CORE CONCEPTS

Questions 13-19

16.

(A) The spring mathematics orders are in folder I.


(B) The fall oceanography orders are in folder I.
(C) The fall nutrition orders are in folder 4, and the
fall oceanography orders are in folder 6.
(D) The fall oceanography orders are in folder 2, and
the spring oceanography orders are in folder I.
(E) The spring oceanography orders are in folder I,
and neither the spring nor the fall nutrition orders
are in folder 3.

A college offers one course in each of three subjects


mathematics, nutrition, and oceanography-in the fall and
again in the spring. Students' book orders for these course
offerings are kept in six folders, numbered 1 through 6, from
which labels identifying the folders' contents are missing. The
following is known:
Each folder contains only the orders for one of the six
course offerings.
Folder 1 contains orders for the same subject as folder 2
does.
The orders in folder 3 are for a different subject than are
the orders in folder 4.
The fall mathematics orders are in folder I or else
folder 4 .
The spring oceanography orders are in folder I Or else
folder 4.
The spring nutrition orders are not in folder 5.

13.

Which one of the following could be the list of the


contents of the folders, in order from folder I to
folder 6?
(A) fall mathematics, spring mathematics, fall
oceanography, fall nutrition, spring nutrition,
spring oceanography
(8) fall oceanography, spring nutrition, fall nutrition,
fall mathematics, spring mathematics, spring
oceanography
(C) spring mathematics, fall mathematics, spring
nutrition, fall oceanography, fall nutrition, spring
oceanography
(D) spring oceanography, fall oceanography, fall
nutrition, fall mathematics, spring mathematics,
spring nutrition
(E) spring oceanography, fall oceanography, spring
mathematics, fall mathematics, fall nutrition,
spring nutrition

14.

Which one of the following statements must be false?


(A)
(8)
(C)
(D)
(E)

15.

The
The
The
The
The

spring mathematics orders are in folder 3.


fall nutrition orders are in folder 3.
spring oceanography orders are in folder 1.
spring nutrition orders are in folder 6.
fall oceanography orders are in folder 5.

If the fall oceanography orders are in folder 2, then


which one of the following statements could be true?
The spring mathematics orders are in folder 4.
The spring mathematics orders are in folder 6.
The fall nutrition orders are in folder I.
The spring nutrition orders are in neither folder 3
nor folder 6 .
(E) Neither the spring nor the fall nutrition orders are
in folder 3 .
(A)
(8)
(C)
(D)

142

The Princeton Review. Inc

Which one of [he following statements could be true?

17.

If the fall oceanography orders are in folder 2, then for


exactly how many of the remaining five folders can it
be deduced which course offering's orders are in that
folder?
(A)
(B)
(C)
(D)
(E)

18.

Which one of the following lists a pair of folders that


must together contain orders for two different subjects?
(A)
(B)
(C)
(D)
(E)

19.

one
two
three
four
five

3 and 5
4 and 5
3 and 6
4 and 6
5 and 6

Which one of the following could be true?


(A) The fall mathematics and spririg oceanography
orders are in folders with consecutive numbers.
(8) Folder 5 contains the orders for a spring course in
a subject other than mathematics.
(C) Folder 6 contains the orders for a subject other
than nutrition.
(D) The mathematics orders are in folders I and 4 .
(E) The orders for the fall courses are in folders I , 3,
and 6.

UNIT 4

QJJestions 20 24

22 .

In a theater company, four two-day workshops-Lighting,


Production , Rehearsals, and Staging-are conducted over the
course of five days, Monday through Friday. The workshops
are conducted in a manner consistent with the following
constraints :
The two days on which a given workshop is in session are
consecutive.
On each of the five days, at least one, but no more than
two, of the workshops are in session.
The workshops on Production and Rehearsals begin no
earlier than the day immediately following the second
day of the workshop on Lighting.
20. Which one of the following could be true ')

(A) The workshop


Monday.
(8) The workshop
Wednesday.
(C) The workshop
Thursday.
(0) The workshop
Monday.
(E) The workshop
Wednesday.

on Rehearsals is in session on
on Staging is in session on
on Staging is in session on
on Staging is in session on

(A) The workshop on Lighting is in session both on


Tuesday and on Wednesday.
(8) The workshop on Rehearsals is in session both on
Wednesday and on Thursday.
(C) The workshop on Staging is in session both on
Monday and on Tuesday.
(0) The workshop on Lighting is in session on the
same two days as is the workshop on Staging.
(E) The workshop on Rehearsals is in session on a
day when the workshop on Staging is also in
session.

Which one of the following could be true')


(A) The workshop on Lighting is in session on
Wednesday, and the workshop on Rehearsals is
in session on Tuesday.
(8) The workshop on Lighting is in session on
Wednesday, and the only workshop in session on
Thursday is the workshop on Rehearsals.
(C) The workshop on Lighting is in session on
Wednesday, and the only workshop in session on
Monday is the workshop on Staging.
(0) The workshop on Lighting is in session on
Monday, and the only workshop in session on
Thursday is the workshop on Staging.
(E) The workshops on Lighting and Production are
both in session on Wednesday.

on Lighting is in session on

23 . If the workshop on production is the only workshop in


session on Friday, which one of the following must be
false?

(A) Only one workshop is in session on Thursday.


(8) Only one workshop is in session on Friday.
(C) The workshop on Rehearsals is in session on
Tuesday.
(0) The workshop on Staging is in session on
Thursday.
(E) The workshops in Rehearsals and Production are
both in session on Wednesday.
21.

If the workshop on Production is in session on


Wednesday, which one of the following must be true')

24.

If the workshop on Lighting is the only workshop in


session on Monday, which one of the following could
be true?
(A) The workshops on Rehearsals and Staging are

both in session on Tuesday.

(8) The workshop on Rehearsals is the only workshop


in session on Wednesday.
(C) The workshop on Staging is the only workshop in

session on Wednesday.
The workshops on Staging and Rehearsals are
both in session on Wednesday and on Thursday.
(E) The workshops on Staging and Production are
both in session on Thursday.

(0)

The Princeton Review. Inc.

I 143

UNIT 5

Group Games~ Fixed Assignments


Qyestions 1-5

3.

There are exactly five pieces of mail in a mailbox: a flyer,


a letter, a magazine, a postcard, and a survey. Each piece
of mail is add ressed to exactly one of three housemates:
Georgette, Jana, or Rini. Each housemate has at least one of
the pieces of mail addressed to her. The following conditions
must apply :
Neither the letter nor the magazine is addressed to
Georgette.
If the letter is addressed to Rini. then the postca rd is
addressed to Jana.
The housemate to whom the flyer is addressed has at least
one of the other pieces of mail addressed to her as well.
1.

(B)

(C)

(D)

(E)

2.

(A)
(B)
(C)
(D)
(E)
4.

Georgette: the flyer, the survey


J ana: the letter
Rini : the magazine
Georgette: the flyer, the pos tcard
1ana: the letter, the magazine
Rini: the survey
Georgette: the magazine, the survey
lana: th e fl yer, the letter
Rini: the postcard
Georgette: the survey
lana: the flyer, the magazine
Rini: the letter, the postcard
Georgette: the survey
lana: th e letter, the magazine, the postcard
Rini: the flyer

5.

the
the
th e
the
the

fly er,
flyer,
flyer,
flyer,
flyer,

the
the
the
the
the

letter, the magazi ne


letter, the postcard
letter, the survey
magazine, the postcard
magazine, the survey

Which one of the following CANNOT be a complete


and accurate list of the pieces of mail addressed to
Rini?
(A)
(B)
(C)
(D)
(E)

Which one of the following could be a complete


and accurate matching of the pieces of mail to the
housemates to whom they are addressed?
(A)

Whic h one of the following CANNOT be a complete


and accurate li st of the pieces of mail addressed
to lana?

the
the
the
the
the

magazine, the postcard


letter, the survey
letter, the magazine
flyer, the magazine
flyer, the letter

If the magazine and the survey are bo th addressed to the


same housemate, then whic h one of th e fo llowing co uld
be true?
(A)
(B)
(C)
(D)
(E)

The
The
The
The
The

survey is addressed to Georgette.


postcard is addressed to Rini .
magazine is addressed to lana .
letter is addressed to Rini .
fl yer is addressed to l ana.

Which one of the following is a comp lete and accurate


list of the pieces of mail , anyone of which could be the
only piece of mail addressed to lana?
(A)
(B)
(C)
(D)
(E)

the
the
the
the
the

postcard
letter, the postcard
letter, the survey
magazine, the survey
letter, the magazine, the postcard

The Princeton Review, Inc.

/ 157

UNIT 5

~tions

6-12

A total of six books occupies three small shelves-one on


the first shelf, two on the second shelf, and three on the third
shelf. Two of the books are grammars-one of Farsi, the other
of Hausa. Two others are linguistics monographs-one on
phonology, the other on semantics. The remaining two books
are novels-one by Vonnegut, the other by Woolf. The books'
arrangement is consistent with the following:
There is at least one novel on the same shelf as the Farsi
grammar.
The monographs are not both on the same shelf.
The Vonnegut novel is not on the same shelf as either
monograph.
6.

Which one of the following CANNOT be true?


(A) A grammar is on the first shelf.
(B) A linguistics monograph is on the same shelf as
the Hausa grammar.
(C) A novel is on the first shelf.
(D) The novels are on the same shelf as each other.
(E) Neither linguistics monograph is on the first shelf.

Which one of the following must be true?


(A) A linguistics monograph and a grammar are on
the second shelf.
(B) A novel and a grammar are on the second shelf.
(C) At least one linguistics monograph and at least
one grammar are on the third shelf.
(D) At least one novel and at least one grammar are
on the third shelf.
(E) At least one novel and at least one linguistics
monograph are on the third shelf.

9.

If both grammars are on the same shelf, which one of

the following could be true ?

(A)
(B)
(C)
(D)
(E)

Which one of the following could be an accurate

matching of the bookshelves to the books on each of

them?

(A) first shelf: Hausa grammar


second shelf: semantics monograph, Vonnegut
novel
third shelf: Farsi grammar, phonology monograph ,
Woolf novel
(B) first shelf: semantics monograph
second shelf: Farsi grammar, Vonnegut novel
third shelf: Hausa grammar, phonology
monograph, Woolf novel
(C) first shelf: Vonnegut novel
second shelf: phonology monograph, Farsi
grammar
third shelf: Hausa grammar, semantics monograph.
Woolf novel
(D) first shelf: Woolf novel
second shelf: phonology and semantics
monographs
third shelf: Farsi and Hausa grammars, Vonnegut
novel
(E) first shelf: Woolf novel
second shelf: Farsi grammar, Vonnegut novel
third shelf: Hausa grammar, phonology and
semantics monographs

7.

8.

10.

The phonology monograph is on the third shelf.


A novel is on the first shelf.
Both novels are on the second shelf.
The Farsi grammar is on the second shelf.
The phonology monograph is on the first shelf.

Which one of the following must be true?


(A) A linguistics monograph is on the first shelf.
(B ) No more than one novel is on each shelf.
(C) The Farsi grarrunar is not on the same shelf as the
Hausa grammar.
(D) The semantics monograph is not on the same
shelf as the Woolf novel.
lE) The Woolf novel is not on the first shelf.

11.

If the Farsi grammar is not on the third shelf, which one


of the following could be true?
(A)
(B)
(C)
(D)
(E)

12.

The
The
The
The
The

phonology monograph is on the second shelf.


Hausa grammar is on the second shelf.
semantics monograph is on the third shelf.
Vonnegut novel is on the third shelf.
Woolf novel is on the second shelf.

If the Hausa grammar and the phonology monograph


are on the same shelf, which one of the following must
be true ?
(A)
(B)
(C)
(D)
(E)

The
The
The
The
The

phonology monograph is on the third shelf.


Vonnegut novel is on the second shelf.
semantics monograph is on the second shelf.
semantics monograph is on the first shelf.
Woolf novel is on the third shelf.

The Princeton Review, Inc. 1 159

Group Games: Fixed Assignments


Ouestions 1-5
Each of nine students-Faith, Gregory, Harlan, Jennifer, Kenji,
Lisa , Marcus, Nari, and Paul-will be assigned to exactly one
of three panels: Oceans, Recycling, and Wetlands. Exactly
three of the students will be assigned to each panel. The
assignment of students to panels must meet the following
conditions:
Faith is assigned to the same panel as Gregory.
Kenji is assigned to the same panel as Marcus.
Faith is not assigned to the same panel as Paul.
Gregory is not assigned to the same panel as Harlan.
Jennifer is not assigned to the same panel as Kenji.
Harlan is not assigned to the Oceans panel if Paul is not
assigned to the Oceans panel.
I.

Which one of the following is an acceptable assignment


of students to the panels')
(A) Oceans: Faith, Gregory, Jennifer

Recycling: Kenji , Lisa, Nari

Wetlands: Harlan, Marcus, Paul

(B) Oceans: Faith, Jennifer, Lisa

Recycling: Harlan, Kenji, Marcus

Wetlands: Gregory, Nari, Paul

(C) Oceans: Harlan, Kenji, Marcus

Recycling: Faith, Gregory, Jennifer

Wetlands: Lisa, Nari, Paul

(D) Oceans: Jennifer, Kenji, Marcus

Recycling: Faith, Gregory, Nari

Wetlands: Harlan, Lisa , Paul

(E) Oceans: Kenji, Marcus, Paul

Recycling: Harlan , Jennifer, Nari

Wetlands: Faith , Gregory, Lisa

2.

If Marcus and Paul are both assigned to the Wetlands


panel, which one of the following must be true?
(A) Harlan is assigned to the Recycling panel.
(B) Jennifer is assigned to the Oceans panel.
(C) Kenji is assigned to the Recycling panel.
(D) Lisa is assigned to the Wetlands panel.
(E) Nari is assigned to the Oceans panel.

188 I

The Princeton Review. inc

3.

Which one of the following is a pair of students who


could be assigned to the same panel as each other?
(A) Faith and Harl an
(B) Gregory and Paul
(C) Harlan and Marcus
(D) Faith and Marcus
(E) Jennifer and Marcus

4.

If Kenji and Paul are both assigned to the Recycling


panel, which one of the following could be true?
(A) Faith is assigned to the Wetlands panel.
(B) Gregory is assigned to the Recycling panel.
(C) Harlan is assigned to the Oceans panel.
(D) Jennifer is assigned to the Wetlands panel.
(E) Lisa is assigned to the Recycling panel.

5.

Each of the following is a pair of students who could be


assigned to the same panel as each other EXCEPT:
(A) Gregory and Kenji
(B) Gregory and Lisa
(C) Kenji and Nari
(D) Lisa and Marcus
(E) Lisa and Paul

UNIT 5

Questions 6-11
Each of five students-Hubert, Lori, Paul, Regina , and
Sharon-will visit exactly one of three cities-Montreal,
Toronto, or Vancouver-for the month of March , according to
the following conditions:
Sharon visits a different city than Paul.
Hubert visits the same city as Regina.
Lori visits Montreal or else Toronto.
If Paul visits Vancouver, Hubert visits Vancouver with
him.
Each student visits one of the cities with at least one of
the other four students.
6.

If Hubert and Sharon visit a city together, which one of


the following could be true in March?
(A)
(B)
(C)
(D)
(E)

8.

Hubert visits the same cit y as Paul.


Lori visits the same city as Regina .
Paul visits the same city as Regina.
Paul visits Toronto.
Paul visits Vancouver.

Which one of the following could be false in March?


(A) Sharon must visit Monrreal if Paul visilS
Vancouver.
(B) Regin a must visit Vancouver if Paul vi sits
Vancouvef.
(e) Regina visits a city with exactly two of the other
four students.
(D) Lori visits a city with exactly one of the other
four students.
(E) Lori visits a city with Paul or else with Sharon.

10.

Which one of the following could be true for March?


(A) Hubert, Lori, and Paul vi sit Toronto, and Regina
and Sharon visit Vancouver.
(B ) Hubert, Lori, Paul , and Regina visit Montreal , and
Sharon visits Vancouver.
(C) Hubert, Paul , and Regina visit Toronto, and Lori
and Sharon visit Montreal.
(D) Hubert, Regina, and Sharon visit Montreal, and
Lori and Paul visit Vancouver.
(E) Lori, Paul , and Sharon visit Montreal, and Hubert
and Regina visit Toronto.

7.

9.

If Regina visits Toronro, which one of lhe following


could be true in March?
(A)
(B)
(e)
(D)
(E)

11 .

Lori visilS Toronto.


Lori visits Vancouver.
Paul visits Toronto
Paul visits Vanco uver.
Sharon visits Vancouver.

Which one of the following must be true fOf March ?

If any of the students


Montreal
(B) If any of the student s
of them do.
(e ) If any of the studenrs
of them do.
(D) If any of the students
Vancouver.
(E) If any of the students
three of them do.
(Al

visits Montreal, Lori visits


visits Montreal, exactly two
visits Toronto, exactly three
visits Vancouver, Paul vi si ts
visits Vancouvef, exactly

If Sharon visits Vancouver, which one of the following


must be true for March ?
(A)
(B)
(C)
(D )
(E)

Hubert visits Montreal.


Lori visits Montreal
Paul visits Toro nto.
Lori visits the same city as Paul.
Lori visits the same city as Regina.

The Pr;nce ton Rev;ew, Inc.

1189

LSAT 101: CORE CONCEPTS

14.
camp counselors-Fran,
Lewis, Nathan, and
tennis, and
one of three
counselors must conform to the
volleybalL The
following conditions:
Each

12.

Fran and Kathy

tennis.
the same activity as

tennis.

volleyball.

volleyball.

In'''''r\t~Cp.C' tennis.

Which one of the


is an acceptable
of the counselors to the activities')
Fran, George,
Tennis: Joan,
Lewis; Volleyball: Kathy, Nathan,
Swimming:
Tennis: Fran,
Joan, Lewis; Volleyball:
Nathan
eC)
Henry; Tennis: Fran,
Joan,
Lewis; Volleyball: Kathy, Nathan, Olga
(D)
Joan, Kathy; Tennis: George,
Fran. Lewis,
(E)
Nathan; Tennis: Fran. Kathy,
Lewis; Volleyball:
Joan, Olga

13.

If George and Kathy are two of three counselors


_
to
which one of the
following could be true of the
(Al Fran supervises

both Nathan and

(A)

Which one of the following is a


of counselors who
could be two of three counselors assigned to supervise

16.

If Fran and Lewis are two of three counselors


to
swimming, which one of the following
must be true of the
CA)

volleyball.

Henry supervises volleyball.

(e) Joan
(D) Kathy
(E) Nathan
17.

If Joan is
to
the same activity as
which one of the following CANNOT be true of
the

(A)

(8)
Joan and
(D) Joan and ;-.Iathan
Joan and

190

The PnncelOo Review. Inc

tennis')

(8) George and Nathan

15.

Neither Kathy nor


Neither Kathy nor Nathan
Joan.
If

Which one of the following is a


could
be assigned to

(E) Nathan supervises tennis.

UNIT5

Qu estions 18- 23

20.

A new ly for med compan y has five employees- F, G , H, K,


and L. Eac h employee h olds exactl y one of the fo ll owing po
sitions: pres ide nt, manager, or tech nicia n. Only the president
is not sup erv ised . Other emplo yees are e ach supervised by
exactly o ne employee, who is eith er th e pres id ent or a man
ager. E ach supervi sed employee holds a d iffe re nt position
th an hi s or her s upervisor. The fo llowing conditio ns a pply:
There is exactly one presid ent.

At least one of the employees whom the president

sup er vises is a manager.


Each manager supervises at leas t one employee.
F does not supervise any employee.
G sup ervises exactly two employees.
18.

(A)
(B)
(C)
(D )
(E)

19 .

(A )
(B)
(C)
(D )
(E)

2 1.

M anager

G
G

H, K,L

H
H, K
K

F, G
G
F, G,

Technic ian
F
F, K,

F, H
F, L
G,K
G, L
K, L

Whic h one of the foll owing could be true?


(A) T here is exactl y one tec hnician.
(B) There are exactl y two managers.
(C) Th ere are exactly two employees who are not
supervised .
(D) T here are more managers th an technician s.
(E) T he president supervises all of th e other
employees .

Which one of the follow ing is an acceptable ass ignment


of employees to th e positions?
President

Which one of th e following is a pair of e mployees who


could serve as man agers together?

22.

If F is superv ised by th e presid ent, which one of th e


followi ng must be uue?
(A)
(B)
(C)
(D)
(E)

K, L
FL
H, L

G is th e president.
H is the pres id ent.
L is a technician.
There is exactl y one manager.
There are exactl y two technician s.

Wh ich one of the following must be true?


(A)
(B)
(C)
(D )
(E)

There are at most three technici ans.


There is exactly one technician.
There are at least two managers.
There are exactl y two man agers.
T here are exactly two employees who su pervise
no one.

23.

If K supervises exactl y two employees, which one of


the following mu st be true?
(A)
(B)
(C)
(D )
(E)

F is supervised by K.
G is a ma nager.
L is supervis ed.
There are ex actl y two managers.
There are exactl y two techni cians.

The P(inCelOn ReView, Inc. I

191

lSAT 101 : CORE CONCEPTS

Games: Determining the Best Diagram


Questions 1-S

3.

Exactly six of an artist's paintings, entitled Quarterion,


Redemption, Sipapu, Tesseract, Vale, and Zelkova, are sold
at auction. Three of the paintings are sold to a museum, and
three are sold to a private collector. Two of the paintings are
from the artist's first (earliest) period, two are from her second
period, and two are from her third (most recent) period. The
private collector and the museum each buy one painting from
each period. The following conditions hold:
Sipapu, which is sold to the private collector, is from
an earlier period than Zelko va , which is sold to the
museum.
Quarterion is not from an earlier period than Tesseract.
Vale is from the artist's second period.
1.

2.

If Sipapu is from the artist's second period, which one

of the following could be two of the three paintings

bought by the private collector?

(A) Quarterion and Zelkova


(B) Redemption and Tesseract
(e) Redemption and Vale
(D) Redemption and Zelkova
(E) Tesseract and Zelkova

212

(Al Quarterion, Redemption


(B) Redemption, Sipapu
(e) Quarterion, Sipapu, Tesseract

(D) Quarterion, Redemption, Sipapu, Tesseract


(E) Redemption, Sipapu, Tesseract, Zelko va
4.

I The Princeton ReView, Inc.

If Sipapu is from the artist's second period, then which


one of the following paintings could be from the period
immediately preceding Quarterion's period and be sold
to the same buyer as Quarterion?
(A)
(B)
(e)
(D)
(E)

Which one of the following could be an accurate list

of the paintings bought by the museum and the private

collector, listed in order of the paintings' periods, from

first to third?

(A) museum: Quarterion, Vale, Zelkova


private collector: Redemption, Sipapu, Tesseract
(B) museum: Redemption, Zelkova, Quarterion
private collector: Sipapu, Vale, Tesseract
(e) museum: Sipapu, Zelkova, Quarterion
private collector: Tesseract, Vale, Redemption
(D) museum: Tesseract, Quarterion, Zelko va
private collector: Sipapu, Redemption, Vale
(E) museum: Zelkova, Tesseract, Redemption
private collector: Sipapu, Vale, Quarterion

Which one of the following is a complete and accurate


list of the paintings, anyone of which could be the
painting from the artist's first period that is sold to the
private collector?

S.

Redemption
Sipapu
Tesseract
Vale
Zelkova

If Zelkova is sold to the same buyer as Tesseract and


is from the period immediately preceding Tesseract's
period, then which one of the following must be true?
(A) Quarterion is sold to the museum.
(B) Quarterion is from the artist's third period.
(e) Redemption is sold to the private collector.
(D) Redemption is from the artist's third period.
(E) Redemption is sold to the same buyer as Vale.

UNIT6

Ouestions 6-11
A swim team with exactly five members-Jacobson, Kruger,
Lu, Miller, Ortiz-swims a ten-lap relay race. Each team
member swims exac tly two of the laps: one swims laps 1 and
6, one swims laps 2 and 7, one swims laps 3 and 8, one swims
laps 4 and 9, and one swims laps 5 and 10. The following
conditions apply:
Neither of Kruger's laps is immediately before either of
Lu's.
Jacobson does not swim lap 9.
Ortiz's first lap is after (b ut not necessarily immediately
after) Mi lIer's.
At least one of Jacobson's laps is immediately after one
of Ortiz's laps.
6.

Which one of the following could be an accurate list of


the swimmers of the first five laps, in order from lap I
through lap 5 ?

8.

(A)
(B)
(C)
(D)
(E)
9.

If Jacobson swims lap 8, then for exactly how many


of the ten laps can one detennine which team member
swims the lap?
(A) ten
(8) eight
(C) six
(D) four
(E) two

Jacobson swims lap 1.


Jacobson swims lap 3.
Kruger swims lap 5.
Lu swi ms lap 3.
Miller swims lap 5.

Which one of the following could be true?


(A) Jacobson swims lap 4.
(8) Kruger swims lap 5.

(C) Lu swims lap 5.


(D) Miller swi ms lap 10.
(E) Ortiz sw ims lap 6.
10.

Jacobson CANNOT swim which one of the following


laps?
(A) lap
(8) lap
(C) lap
(D) lap
(E) lap

(A) Jacobson, Kruger, Miller, Lu , Ortiz


(8) Kruger, Miller, Ortiz, Jacobson , Lu
(C) Lu, Miller, Jacobso n, Kruger, Ortiz
(D) Ortiz, Kruger, Miller, Lu , Jacobson
(E) Miller, Ortiz, Jacobson, Kruger, Lu
7.

If Ortiz swim s lap 4, then which one of the following


could be true?

11.

I
2
3
6
10

Which one of the following could be an accurate list of


the swimmers of the last five laps , in order from lap 6
through lap 10 ?
(A)
(8)
(C)
(D)
(E)

Jacobso n, Miller, Kruger, Ortiz, Lu


Kruger, Lu, Miller, Ortiz, Jacobson
Lu , Kruger, Miller, Ortiz, Jacobson
Miller, Kruger, Ortiz, Jacobson , Lu
Ortiz, Jacobson, Kruger, Miller, Lu

The Princelon Review. Inc.

213

UNIT6

Games: Mixed Bag


Ouestions 1-5
Doctor Yamata works only on Mondays, Tuesdays , Wednes
days , Fridays, and Saturdays. She performs four differ
ent activities-lecturing. operating , treating patients. and
conducting research. Each working day she performs exactly
one activity in the morning and exactly one activity in the
afternoon . During each week her work schedule mu s t satisfy
the following restrictions:
She performs operatiom on exactly three mornings.
If she operates on Monday, she does not operate on
Tuesday.
She lectures in the afternoon on exactly two consecutive
calendar days.
She treats patients on exactly one morning and exactly
three afternoons.
She conducts research on exactly one morning.
On Saturday she neither lectures nor performs operations .
1.

4.

(A) conduct research in the morning and operate in


the afternoon
(8) lecture in the morning and treat patients in the
afternoon
(C) operate in the morning and lecture in the
afternoon
(D) operate in the morning and conduct research in
the afternoon
(E) treat patients in the morning and treat patients in
the afternoon

If Doctor Yamata operates on Tuesday, then her


schedule for treating patients could be
(A) Monday morning, Monday afternoon , Friday
morning, Friday afternoon
(8) Monday morning, Friday afternoon, Saturday
morning, Saturday afternoon
(e) Monday afternoon, Wednesday morning,
Wednesday afternoon , Saturday afternoon
(D) Wednesday morning, Wednesday afternoon, Friday
afternoon , Saturday afternoon
(E) Wednesday afternoon , Friday afternoon , Saturday
morning, Saturday afternoon

Monday
Tuesday
Wednesday
Friday
Saturday

On Wednesday Doctor Yamata could be scheduled to

Which one of the following statements must be true?


(A) There is one day on which the doctor treats
patients both in the morning and in the
afternoon.
(8) The doctor conducts research on one of the days
on which she lectures.
(e) The doctor conducts research on one of the days
on which she treats patients.
(D) The doctor lectures on one of the days on which
she treats patients.
(E) The doctor lectures on one of the day s on which
she operates.

Which one of the following must be a day on which

Doctor Yamata lectures?

(A)
(8)
(C)
(D)
(E)

2.

3.

5.

Which one of the following is a pair of days on both of


which Doctor Yamata must treat patients?
(A) Monday and Tuesday
(8 ) Monday and Saturday
(e) Tuesday and Friday
(D) Tuesday and Saturday
(E) Friday and Saturday

The Princeton ReView, Inc

I 231

LSAT 101: CORE CONCEPTS

9.

Questions 6-12
A florist is making three corsages from four types of flowers:
gardenias, orchids, roses, and violets. Each of the corsages
will contain exactly three flowers. The nine flowers used in
the corsages must include at least one flower from each of the
four types, and at least twice as many roses as orchids must
be used. The corsages must also meet the following specifica
tions:
Corsage 1 must contain exactly two types of flowers.
Corsage 2 must contain at least one rose.

Corsage 3 must contain at least one gardenia but no

orchids.

6.

(A)

7.

Corsage 2

Corsage 3

2 gardenias
1 rose

1 orchid
1 rose
1 violet

I gardenia
1 orchid
1 violet

(8)

2 orchids
1 rose

2 orchids
I rose

2 gardenias
1 rose

(C)

2 orchids
1 rose

3 roses

I gardenia
2 violets

(D)

1 gardenia
J orchid
rose

1 gardenia
I rose
J violet

1 gardenia
I rose

I violet

(E)

orchid
2 roses

3 violets

3 gardenias

The maximum total number of roses that can be used in


the three corsages is
(A) three
(8) four

(C) five
(D) six

(E) seven

8.

If corsage I contains two orchids and one rose, what is

the maximum total number of violets that the florist can

use in making the three corsages?

(A) one
(8) two

(C) three
(D) four
(E) fi ve

232 I

(A) two orchids


(8) three gardenias

(C) three roses


(D) fi ve roses

(E) five violets

10.

The Princeton Revie w, Inc.

If two of the corsages contain at least one orchid each .


then the flowers in corsage 2 must include at least
(A) one gardenia and one orchid
(B) one gardenia and one rose

(C) one orchid and one rose


one orchid and one violet

(El one rose and one violet

(D)

Which one of the following is an acceptable selection of


flowers for the three corsages?
Corsage I

If corsage 2 is exactl y the same as corsage 3, the ni ne


flowers used in the corsages can include exactly

11.

If the greatest possible number of violets is used in the


three corsages, the florist must use
(A) exactly one rose and exactly one gardenia
(8) exactly one orchid and exactly four violets

(Cl exactly two orchids


(D) exactly two roses
(E) exactly six violets

12.

If corsage I contains at least one gardenia and at least


one violet, and if corsage 3 contains three different
types of flowers, which one of the following could be
used to make corsage 2?
(A)
(8)
(C)
(D)
(E)

one
one
one
two
two

rose, one orchid, and one gardenia


rose and two orchids
rose and two violets
roses and one gardenia
roses and one violet

UNIT 6

Questions 13-17
Three boys-Karl, Luis, and Miguel-and three girls-Rita,
Sarah, and Tura-are giving a dance recital. Three dances
1, 2, and 3-are to be performed. Each dance involves three
pairs of children, a boy and a girl partnering each other in
each pair, according to the following conditions:
Karl partners Sarah in either dance 1 or dance 2.
Whoever partners Rita in dance 2 must partner Sarah in
dance 3.
No two children can partner each other in more than one
dance.
13 .

If Miguel partners Rita in dance 2, which one of the


following could be true?
(A) Karl partners TUfa in dance 1.
(B) Luis partners Sarah in dance 2.
(e) Luis partners Sarah in dance 3.
(D) Miguel partners Sarah in dance 1.
(E) Miguel partners Tura in dance 3.

If Miguel partners Sarah in dance 1, which one of [he


following is a pair of children who must partner each
other in dance 3?
(A) Karl and Rita
(B) Karl and Tura
(e) Luis and Rita
(D) Luis and Tura
(E) Miguel and Tura

16.

If Sarah partners Luis in dance 3, which one of the


following is a complete and accurate list of the girls any
one of whom could partner Miguel in dance 1?
(A) Rita
(B) Sarah
(e) Tura
(D) Rita, Sarah
(E) Rita, Tura

14.

IS.

If Luis partners Sarah in dance 2, which one of the


following is a pair of children who must partner each
other in dance 1?
(A) Karl and Rita
(B) Karl and Tura
(e) Luis and Rita
(D) Luis and Tura
(E) Miguel and Rita

17 .

If Miguel partners Rita in dance 1, which one of the


following must be true?
(A) Karl partners
(B) Karl partners
(e) Karl partners
(D) Luis partners
(E) Luis partners

Rita in dance 2.
Sarah in dance 3.
Tura in dance 1.
Rita in dance 2.
Tura in dance 3.

The Princeton Review, Jnc.

! 233

lSAT 101 GORE CONCEPTS

20.
On the b asis of an examination, nIne
Ida, Jan,
Laura, Mike, and Nick-are each
in one of three classes. The three highest scorers are
in the level 1 class: the three lowest scorers are
in the level 3 class. The
three are
in the
level 2 class. Each class has exactly three students.
Ida scores higher than Glen.
Glen scores
than both Jan and Kathy.
Jan scores higher than Mike,
Mike scores higher than Hilary,
Hilary scores
than Nick.
Kathy scores hi2her than both Fred and Laura.
18.

19.

Which one of the following students could be in the


level 2 class but cannot be in the level 3 class?
Fred

Glen

Jan

(E) Nick

234

(A) Fred
(B) Jan

Kathy

Laura

(E) Mike

21. The composition of each class can be completely

determined jf which one of the following pairs of

students is known to be in the level 2 class?

Fred and
(B) Fred and Mike

and Jan

and Laura

(E) Laura and Mike

How many different combinations of students could


form the level 1 class?
(A) one

two

(C) three
(D) four

(El six

I The Princeton Review, Inc

Which one of the


students could be placed in
anyone of the three classes"

22.

Which one of the following


in the same class as Fred?
Hilary and Nick

Jan and Laura

(C) Kathy and Laura


Jan and Mike

(El Laura and Mike

of students cannot be

UNfT6

25 ,

Questions 23-28
A soloist will play six different guitar concertos, exactly one
each Sunday for six con secutive weeks. Two concertos will be
selected from among three concertos by Giuliani-H, J, and
K; two from among four concertos by Rodrigo-M , N, 0, and
P; and two from among three concertos by Vivaldi -X , Y,
and Z. The following conditions apply without exception:
If N is selected , th en J is a lso selected .

If M is selec ted , then neither J nor 0 can be selected .

If X is selected , then neither Z nor P can be selected.

If both J and 0 are selected, then J is played at some

time before O.
X ca nnot be played on the fifth Sunday unless one of
Rodrigo's concertos is played on the first Sund ay.

If J, 0 , and Yare the first three concertos to be pl ayed ,


not necessaril y in the order given , which one of the
following is a concerto that CANNOT be pl ayed on the
fi fth Sunday')

(A) H
(8 ) K

(C ) N
(D ) P

(E) X
26.

If 0 is selected for the first Sunday, which one of the


following is a conce rto th at must also be selected?
(A) J

(8 ) K
(C) M

23 .

Which one of the followin g is an acceptable selection of


concertos that the soloist could play on the first through
the sixth Sunday,)

.1

1.

5.

M
Y

N
0
J
H
0

Y
Z

K
N
M
0
Z

(A)

(8 )

K
K
p
X

(C)
(D )

(E)
24.

Y
Y
N

P
J
K

Z
X
J

(D) N
(E) X
27 .

Which one of the following is a concerto that must be


selected ?
(A) J
(8 ) K

(C) 0
(D) Y
(E) Z

If the six concertos to be played are J , K, N, 0 , Y. and


Z and if N is to be played on the first Sunday, then
which one of the following concertos CANNOT be
played on the seco nd Sunday ')
(A ) J
(8 ) K
(C) 0

(D) Y
(E)

28.

Which one of the following is a concerto that CANNOT


be selected together with N?
(A ) M
(8) 0
(C) P

(D) X
(E) Z

The Prin ce ton Revi ew, Inc

I 235

S-ar putea să vă placă și